Warum ist Impuls keine Funktion der Position in der Quantenmechanik?

In der Quantenmechanik der einheitliche Zeitübersetzungsoperator U. ^ ( t 1 , t 2 ) U. ^ ( t 1 , t 2 ) wird definiert durch U. ^ ( t 1 , t 2 ) | ψ ( t 1 ) = | ψ ( t 2 ) U. ^ ( t 1 , t 2 ) | ψ ( t 1 ) = | ψ ( t 2 ) und der Hamilton-Operator H. ^ ( t ) H. ^ ( t ) ist definiert als die Grenze von ich U. ^ ( t , t + Δt ) - 1 Δt ich U. ^ ( t , t + Δ t ) - - 1 Δ t wie Δt Δ t geht zu 0 0 . In ähnlicher Weise wird der eindimensionale räumliche Übersetzungsoperator durch definiert T. ^ ( x 1 , x 2 ) | x 1 = | x 2 T. ^ ( x 1 , x 2 ) | x 1 = | x 2 und der Impulsoperator p ^ p ^ ist definiert als die Grenze von ich T. ^ ( x , x + Δ x ) - 1 Δ x ich T. ^ ( x , x + Δ x ) - - 1 Δ x wie Δ x Δ x geht zu 0 0 . Meine Frage ist, warum der Hamilton-Operator eine Funktion des Zeitparameters sein kann t t Der Impulsoperator kann jedoch keine Funktion des Positionsparameters sein x x ?

Die einzige gute Antwort, die ich auf diese Frage erhalten habe, ist, dass die Zeit kein Operator in der nicht-relativistischen Quantenmechanik ist, während die Position ein Operator ist, sodass der Impuls als Funktion der Position die Kommutierungsbeziehung zwischen Position und Impuls beeinträchtigen würde. Aber diese Erklärung macht für mich keinen Sinn, weil man den Fall des Spin-Drehimpulses betrachtet. Wenn R. ^ z ( θ 1 , θ 2 ) R. ^ z ( θ 1 , θ 2 ) bezeichnet den Rotationsoperator für Eigenrotationen um die z-Achse (im Gegensatz zu Orbitalrotationen), dann den Spin-Drehimpulsoperator J. ^ z J. ^ z (im Gegensatz zu Beyoncé) ist definiert als die Grenze von ich R. ^ z ( θ , θ + Δθ ) - 1 Δθ ich R. ^ z ( θ , θ + Δ θ ) - - 1 Δ θ wie Δθ Δ θ geht zu 0 0 . Und doch J. ^ z J. ^ z ist keine Funktion des Winkels θ θ , obwohl es in der Quantenmechanik keinen entsprechenden Operator gibt θ θ . (Es gibt einen anderen Operator namens θ ^ θ ^ , das ist einer der Positionsoperatoren in sphärischen Koordinaten, aber das hat nichts mit Spin und dem zu tun θ θ das ist das, worüber ich spreche.) "Der Parameter hat einen entsprechenden Operator" scheint also nicht die richtige Erklärung zu sein, da es nicht erklärt, warum der Drehimpuls keine Funktion des Winkels sein kann.

Beachten Sie, dass ich nicht nach einer Ad-hoc-Erklärung wie "Das wäre physikalisch nicht sinnvoll, wenn es darum geht, wie Energie und Dynamik klassisch funktionieren" suche. Ich möchte eine erste Erklärung der Prinzipien in der Quantenmechanik.

Diese Frage enthält viele Missverständnisse. Der Übersetzungsoperator ist definiert als T. a ) | x⟩ = | x + a⟩ T. ( ein ) | x = | x + ein , wo a R. ein R. . Der Impulsoperator ist definiert als T. ' ( 0 ) T. ' ( 0 ) , so ist es offensichtlich unabhängig von x x : nicht einmal vor der Einnahme a 0 ein 0 tat T. T. darauf ankommen x x . Die Position ist kein Parameter, sondern der Eigenwert eines Operators (und daher ist es nicht sinnvoll zu behaupten, dass ein zweiter Operator davon abhängen kann oder nicht). Das Momentum hängt auch nicht von der Position in der klassischen Mechanik ab, daher ist unklar, warum es im QM sein sollte.
In ähnlicher Weise ist der Rotationsoperator definiert als R. ^ ( α ) | x = | R ( α ) x R. ^ ( α ) | x = | R. ( α ) x , wo R ( α ) R. ( α ) bezeichnet eine Rotationsmatrix um α ^ α ^ mit Winkel | α | | α | . Der Drehimpulsoperator ist definiert als R. ^ ( 0 ) R. ^ ( 0 ) , so ist es offensichtlich unabhängig von θ θ : nicht einmal vor der Einnahme α 0 α 0 hing es davon ab θ θ . Es gibt auch einen Operator, mit dem identifiziert werden kann θ ^ θ ^ und das Paar J. ^ ich , θ ^ j J. ^ ich , θ ^ j verhält sich sehr ähnlich wie p ^ ich , x ^ j p ^ ich , x ^ j (obwohl das Bild subtiler ist).
@AccidentalFourierTransform Die Frage ist jedoch, warum der Operator für räumliche Übersetzer nicht davon abhängt x x , während der Zeitübersetzungsoperator davon abhängen kann t t ? Und die Antwort kann nicht sein, dass die Position ein Operator in der nichtrelativistischen Quantenmechanik ist, während die Zeit dies nicht ist, weil sie nicht erklärt, warum der intrinsische Rotationsoperator nicht davon abhängt θ θ angesichts dessen θ θ ist auch kein Operator.

Antworten (7)

Ihre Vorstellung davon, was in Bezug auf etwas Irreführendes definiert ist. Normalerweise nimmt der Physiker die infinitesimalen Generatoren H. , p , x H. , p , x als gegebene selbstadjunkte Operatoren und definiert die endlichen Transformationen zu sein U. ( t ) = exp ( - i t H. ) , T. ( ξ ) = exp ( i ξ p ) , S. ( π ) = exp ( i π x ) U. ( t ) = exp ( - - ich t H. ) , T. ( ξ ) = exp ( ich ξ p ) , S. ( π ) = exp ( ich π x ) nach dem Satz von Stone . Wenn H. H. ist dann zeitabhängig U. ( t ) U. ( t ) verwandelt sich in die Dyson-Serie für U. ( t , t 0 ) U. ( t , t 0 ) im Interaktionsbild.

Zur Definition des Impulsoperators selbst: Es wird einfach definiert, der Operator mit zu sein [ x , p ] = i [ x , p ]] = ich . Nach dem Stone-von-Neumann-Theorem sind alle möglichen Wege, Operatoren mit dieser Kommutierungsrelation zu realisieren, im Wesentlichen die gleichen wie auf dem L. 2 ( R ) L. 2 ( R. ) , wo x x ist die Multiplikation mit der Variablen und p p ist Differenzierung. Die Kommutierungsbeziehungen kodieren auch die Transformation T. ( ξ ) T. ( ξ ) fungiert als Übersetzung auf die Position und das S. ( π ) S. ( π ) wirkt als Übersetzung auf den Schwung, siehe auch meine Antwort . Aber entscheidend ist, p p ist per Definition ein einzelner fester Operator . Es ist einfach nicht erlaubt, von irgendetwas abhängig zu sein.

Schließlich scheint Ihre Verwirrung im Grunde dadurch zu entstehen, dass Sie all diese Transformationen mit zwei Parametern schreiben, d. H. U. ( t 0 , t 1 ) , T. ( x 1 , x 2 ) U. ( t 0 , t 1 ) , T. ( x 1 , x 2 ) . Nur die zeitliche Entwicklung darf auf diese Weise von zwei Parametern abhängen, und nur im Fall eines zeitabhängigen Hamilton-Operators. Alle anderen Transformationen sind Ein-Parameter-Gruppen wie im Stone-Theorem, die von einem einzelnen selbstadjunkten Operator generiert werden. Dies wird nicht gezeigt, sondern angenommen . Wir gehen davon aus, dass der Rotationsoperator R ( θ 1 , θ 2 ) R. ( θ 1 , θ 2 ) kümmert sich wirklich nur um den Unterschied zwischen den beiden Winkeln, das heißt, es ist wirklich nur eine Funktion R ( θ 1 - θ 2 ) R. ( θ 1 - - θ 2 ) und wir gehen davon aus, dass die Übersetzung T. ( x 1 , x 2 ) T. ( x 1 , x 2 ) ist wirklich gerecht T. ( x 1 - x 2 ) T. ( x 1 - - x 2 ) . Man könnte es anders annehmen, aber das machen wir in der Standardquantenmechanik nicht.

Wir nehmen das für all diese Transformationen an, weil wir das wollen T. ( x 1 , x 2 ) T. ( x 1 , x 2 ) tatsächlich eine (einheitliche) Darstellung der Übersetzungsgruppe sein R. R. , und die R ( θ 1 , θ 2 ) R. ( θ 1 , θ 2 ) eine Darstellung der Rotationsgruppe sein SO ( 3 ) S. Ö ( 3 ) . Und diese Gruppen enthalten nicht die Transformationen "aus dem Winkel drehen" θ 1 θ 1 zu θ 2 θ 2 ", aber" um Winkel drehen θ θ ", so dass der Operator auch nur von der Differenz abhängt und nicht von den Start- / Endpunkten der Transformation.

Der Fall für die Zeitentwicklung ist anders - obwohl man sagen könnte, dass es eine "Zeitübersetzungsgruppe" gibt, wollen wir tatsächlich einen Operator, der die Entwicklung eines dynamischen Systems codiert. Und in einem dynamischen System können wir uns das leicht vorstellen t 0 t 0 etwas ist "ein- / ausgeschaltet", das die Dynamik des Systems nach diesem Punkt verändert, so dass U. ( t 1 , t 2 ) U. ( t 1 , t 2 ) ist unterschiedlich, je nachdem ob beide t 1 , t 2 t 1 , t 2 sind vorher oder nachher t 0 t 0 .

Ich stimme nicht zu Es ist vollkommen Standard, die einheitlichen Operatoren als primitiv zu betrachten und die infinitesimalen Generatoren in Bezug auf diese zu definieren. Das ist es, was unzählige Lehrbücher tun, darunter Townsend und (wenn ich mich recht erinnere) Sakurai. Und dann wird die Kommutierungsbeziehung abgeleitet, nicht angenommen. In ähnlicher Weise wird auch die Dyson-Reihe abgeleitet, nicht angenommen. Im Kontext dieses Ansatzes stelle ich meine Frage.
@KeshavSrinivasan Dann gilt noch mein letzter Absatz: Dass der Übersetzungsoperator T. ( x 1 , x 2 ) T. ( x 1 , x 2 ) hängt eigentlich nur ab von x 1 - x 2 x 1 - - x 2 und ist daher eine Ein-Parameter-Untergruppe, auf die wir den Satz von Stone anwenden können, um einen einzelnen unabhängigen Impulsoperator zu erhalten, ist eine Annahme .
Genau das ist meine Frage: Warum nehmen wir das an? T. ( x 1 , x 2 ) T. ( x 1 , x 2 ) hängt nur ab von x 1 - x 2 x 1 - - x 2 , und R. z ( θ 1 , θ 2 ) R. z ( θ 1 , θ 2 ) hängt nur ab von θ 1 - θ 2 θ 1 - - θ 2 , aber wir erlauben U. ( t 1 , t 2 ) U. ( t 1 , t 2 ) direkt davon abhängen t 1 t 1 und t 2 t 2 ?
@ KeshavSrinivasan Ich habe eine Begründung dafür hinzugefügt.
@KeshavSrinivasan Weil diese Art der Beschreibung physikalischer Systeme funktioniert? Ich bin mir nicht sicher, nach welchem ​​Grund Sie wirklich suchen - am Ende sind die in unseren physikalischen Theorien getroffenen Annahmen da, weil die resultierende Theorie die Ergebnisse von Experimenten korrekt vorhersagt. Auch, naiv, würden Sie nicht zustimmen, etwas in Position zu bringen x 1 x 1 zu x 1 + b x 1 + b und etwas bei x 2 x 2 zu x 2 + b x 2 + b sollte die gleiche Operation sein, egal was passiert x 1 x 1 und x 2 x 2 sind?

Ich denke, die Antwort liegt in der Kausalität. Das typische Problem, das wir in der nichtrelativistischen Quantenmechanik ansprechen, ist "Gegeben eine Anfangsbedingung ψ ( x , t = 0 ) ψ ( x , t = 0 ) , was ist die Wellenfunktion ψ ( x , T. ) ψ ( x , T. ) zu einem späteren Zeitpunkt T. T. ? "Die Tatsache, dass der Hamiltonianer H. H. Das Erzeugen der Zeitübersetzung / -entwicklung darf explizit von der Zeit abhängen. Dies entspricht der Tatsache, dass wir als Experimentatoren das System nach Belieben extern steuern können und der externe Antrieb nicht endogen innerhalb des Systems "vorhergesagt" werden kann. Anders ausgedrückt, die kausalen Einflüsse des externen Antriebs breiten sich rechtzeitig aus, um die zukünftige Wellenfunktion zu beeinflussen.

Aber wenn der Impulsoperator P. P. Das, was Übersetzungen erzeugt, durfte nicht trivial vom Raum abhängen, dann konnten wir in Analogie zur Schrödinger-Gleichung (und auf eine räumliche Dimension vereinfachen) ein Problem mit einer "Anfangsbedingung" aufstellen. ψ ( x = 0 , t ) ψ ( x = 0 , t ) und betrachten Sie das Problem der "Raumentwicklung" der Wellenfunktion in x x nach der Differentialgleichung - ich ψ / x = P. ( x ) ψ ( x ) - - ich ψ /. x = P. ( x ) ψ ( x ) . Wenn der Experimentator frei wäre, sich äußerlich zu verändern P. ( x ) P. ( x ) dann müssten sich die Einflüsse dieser Änderung in einer raumartigen Richtung ausbreiten, um gleichzeitig die Wellenfunktion zu beeinflussen t t aber größer x x Verletzung der Kausalität.

Daher ist die Tatsache, dass die P. P. Der Bediener muss "raumunabhängig" sein, während der H. H. Der Operator kann sich explizit auf die Zeit verlassen. Dies ist eine nicht relativistische Widerspiegelung der Tatsache, dass die vollständige Quantentheorie relativistisch ist und sich kausale Einflüsse nur in zeitlichen Richtungen ausbreiten können. In einem völlig nichtrelativistischen Universum ist der Impulsoperator P. P. wahrscheinlich könnte logisch explizit von der Position abhängen - genau wie Bosonen und Fermionen logischerweise irgendwelche Spins haben könnten, aber in der realen Welt "erben" sie die Spin-Statistik-Beziehung von der zugrunde liegenden relativistischen Theorie.

Was für eine schöne Antwort! Ich hielt buchstäblich in Ehrfurcht inne, als ich las "Aber wenn der Impulsoperator P, der Übersetzungen erzeugt, nicht trivial vom Raum abhängen durfte ........"
Aber warum kann der Drehimpulsoperator dann nicht J. ^ z J. ^ z abhängig vom Winkel θ θ , der Parameter des intrinsischen Rotationsoperators R. ^ z ( θ ) R. ^ z ( θ ) ?
@KeshavSrinivasan Auch dies ist logisch und mathematisch möglich, aber eine solche Abhängigkeit würde explizit Symmetrien aufbrechen, von denen angenommen wird, dass sie für die vollständige relativistische Theorie gelten. Nur als explizite t t Die Abhängigkeit im Hamilton-Operator bricht die zeittranslationale Symmetrie, eine explizite θ θ Abhängigkeit für die J. J. Der Bediener würde die Rotationsinvarianz des Raums brechen - es würde eine "spezielle Richtung" geben, und die Physik könnte sich beispielsweise für Systeme, die dieser Richtung zugewandt sind, anders verhalten als für Systeme, die senkrecht zu dieser Richtung stehen. Es ist völlig logisch möglich, wenn auch unelegant, ...
in einem völlig nichtrelativistischen Universum und kann nicht von ersten Prinzipien ausgeschlossen werden. Da jedoch die spezielle Relativitätstheorie Rotationen und Boosts mischt, würde eine solche Abhängigkeit die gesamte Poincare-Symmetrie der relativistischen Theorie vollständig ruinieren. Wenn der Hamiltonianer ein Subsystem unter der Kontrolle eines Experimentators beschreibt, könnten Sie sogar eine nichttriviale Simulation durchführen θ θ Abhängigkeit durch manuelles Hinzufügen von Energie zum System jedes Mal, wenn es sich nach Norden dreht oder so. Aber es ist technisch schwierig und unnatürlich, eine "fundamentale" relativistische Theorie aufzubauen, die nicht ...
Sehr klar und beredt; Dies sollte die akzeptierte Antwort sein. @ KeshavSrinivasan, ich denke, dass die Antwort auf Ihren Einwand die folgende ist. Wenn der einzige Hamiltonianer, den jeder studierte, der Hamiltonianer war, bei dem das gesamte Universum als das untersuchte System betrachtet wird, dann wäre (1) dieser Hamiltonianer zeitunabhängig und (2) die Zeitübersetzung würde genauso funktionieren wie die räumliche Übersetzung Frage würde verschwinden. Da wir jedoch gerne Systeme untersuchen, die kleiner als das gesamte Universum sind und in denen ein Experimentator die Umgebung kontrollieren kann, behandeln wir die Zeit anders.

Ich denke, die Vorstellung, dass entweder der Positions- oder der Impulsoperator eine Funktion des anderen ist, ist etwas schlecht definiert. Mir ist klar, dass Sie keine Erklärungen aus der klassischen Physik wollen, entschuldigen Sie also bitte im Moment die Analogie zur Hamiltonschen Mechanik.

In der Hamiltonschen Mechanik beschäftigen wir uns mit a 2 n 2 n -dimensionaler Raum, auf dem sich eine 2-Form befinden sollte, dh ein antisymmetrischer 2-Tensor ω i j ω ich j , das sollte nicht entartet sein, ω i j v j 0 ω ich j v j 0 es sei denn v j = 0 v j = 0 und geschlossen, [ i ω j k ] = 0 [ ich ω j k ]] = 0 wobei die Klammern Antisymmetrisierung bedeuten. Diese Bedingungen sind alle koordinatenunabhängig. Schon seit ω i j ω ich j ist nicht entartet, es hat eine Umkehrung ω i j ω ich j . Wenn f g f , G sind Funktionen dazu 2 n 2 n -dimensionaler Raum, können wir eine Operation definieren, die als Poisson-Klammer von bezeichnet wird f f und G G durch

{ f g } = ω i j ( ω ich k k f ) ( ω j l l G ) . { f , G }} = ω ich j ( ω ich k k f ) ( ω j l l G ) .
Beachten Sie, dass die Poisson-Klammer koordinatenunabhängig definiert ist. Das andere, was wir für die Hamiltonsche Mechanik brauchen, ist eine Hamiltonsche Funktion H. H. , das definiert die Dynamik durch
f ˙ = { f , H. } . f ˙ = { f , H. }} .

Die Komponenten der 2-Form ω i j ω ich j sind natürlich koordinatenabhängig. Nach einem Satz von Darboux ist es nun immer möglich, (lokal) sogenannte kanonische Koordinaten zu finden x ich , i = 1 , , 2 n x ich , ich = 1 , , 2 n so dass ω i j ω ich j nimmt die folgende Form an

ω i j = [ 0 - Ich n ich n 0 ]] ω ich j = [ 0 ich n - - ich n 0 ]]
wo ich n ich n ist der n × n n × n Identitätsmatrix. Lassen q ich = x ich , i = 1 , , n ; p ich = x i + n , i = 1 , , n . q ich = x ich , ich = 1 , , n ;; p ich = x ich + n , ich = 1 , , n . Dann können Sie das herausfinden
{ f g } = f q ich G p ich - f p ich G q ich . { f , G }} = f q ich G p ich - - f p ich G q ich .
Im Speziellen,
{ q ich , p j } = δ i j { q ich q j } = { p ich , p j } = 0 { q ich , p j }} = δ ich j { q ich , q j }} = { p ich , p j }} = 0
und ich kann leicht Hamiltons Gleichungen in der kanonischen Form wiederherstellen.

In einem bestimmten Koordinatensystem wie dem beschriebenen, einem kanonischen Koordinatensystem, dem q ich q ich werden die Positionen oder Koordinaten genannt, und die p ich p ich werden die Momente genannt. Keiner ist eine Funktion des anderen. Dies gilt jedoch für jedes Koordinatensystem: Alle Koordinaten sind voneinander unabhängig. Man könnte es genauso gut gebrauchen v ich = p ich - e A. ich ( q j ) v ich = p ich - - e EIN ich ( q j ) als die zweite Hälfte der Koordinaten. Die Komponenten der 2-Form werden komplizierter und die Formel für die Poisson-Klammer wird nicht so schön sein, aber ( x ich , v ich ) ( x ich , v ich ) ist ein perfekt feines Koordinatensystem.

Kommen wir nun zur Quantenmechanik. In der Quantenmechanik haben wir Operatoren, anstatt Kommutatoren anstelle von Poisson-Klammern zu verwenden. Die Dynamik ist gegeben durch

Ö ˙ ^ = i [ O. ^ , H. ^ ]] Ö ˙ ^ = ich [ Ö ^ , H. ^ ]]
und in Analogie zur Hamiltonschen Mechanik führen wir als beobachtbare Operatoren ein x ^ ich , p ^ ich x ^ ich , p ^ ich das befriedigen
[ x ^ ich , p ^ j ] = i δ i j [ x ^ ich , x ^ j ] = [ p ^ ich , p ^ j ] = 0. [ x ^ ich , p ^ j ]] = ich δ ich j [ x ^ ich , x ^ j ]] = [ p ^ ich , p ^ j ]] = 0.
Wir sagen, dass diese Operatoren eine Basis einer Lie-Algebra bilden. Aber so wie ich im klassischen Fall die Koordinaten ändern kann, kann ich im Quantenfall die Basis ändern. Es ist nicht sinnvoll zu sagen, dass der Positionsoperator eine Funktion des Impulsoperators ist oder umgekehrt, da sie alle voneinander unabhängige Basiselemente in der Lie-Algebra sind.

Der Quantenfall kann auch direkt in Form einer Koordinatenänderung formuliert werden. Dann anstelle der Poisson-Halterung auf 2 n 2 n -dimensionaler Phasenraum, man hat die Moyal-Klammer . Wie die Poisson-Klammer wirkt die Moyal-Klammer wie ein Differentialoperator, und der Ausdruck dafür ist in speziellen Koordinatensystemen besonders einfach, es ist jedoch nicht erforderlich, solche Koordinaten zu verwenden.

Dies erklärt jedoch nicht, warum der Drehdrehimpulsoperator keine Funktion des Winkels ist θ θ dass der intrinsische Rotationsoperator R. ^ z R. ^ z ist eine Funktion von. θ θ hat keinen entsprechenden Operator oder Observable, also ist es so t t in dieser Hinsicht. Und doch der Hamilton-Operator H. H. darf eine Funktion von sein t t .
@ KeshavSrinivasan Warum sagst du, dass es keinen Operator für gibt? θ θ und dass es nicht beobachtbar ist? Ist Arccos ( z ^ / r ^ ) Arccos ( z ^ /. r ^ ) nicht gut genug für dich? Denken Sie, dass die Quantenmechanik starre Körper nicht beschreiben kann? (Wie kann dann klassische Mechanik?)
Das ist anders θ θ , wie ich in meiner Frage deutlich gemacht habe. Das θ θ , der einer der Positionsoperatoren in sphärischen Koordinaten ist, ist ein Parameter des Orbitalrotationsoperators. Während die θ θ Ich spreche von einem Parameter des intrinsischen Rotationsoperators, dh des mit dem Spin verbundenen Rotationsoperators. Der Drehimpulsoperator hat eine Kommutierungsbeziehung mit dem θ θ dass du redest. Es gibt jedoch keinen ähnlichen Winkeloperator, zu dem der Spin-Drehimpulsoperator eine Kommutierungsbeziehung hat.
Warum sollte es dann einen solchen Operator geben? Soweit eine klassische Spin-Theorie, dh ein intrinsischer Drehimpuls, existiert, handelt es sich nicht um eine Theorie eines starren Rotors. Siehe zum Beispiel hier physics.stackexchange.com/questions/108387/…
Ja, das ist genau mein Punkt, so ein θ θ Operator existiert nicht. Warum also nicht? J. ^ z J. ^ z eine Funktion von sein θ θ , genauso wie H. ^ H. ^ ist eine Funktion von t t ?

Tatsächlich gibt es Szenarien, in denen der Impulsoperator von der Position abhängen könnte. Betrachten Sie zum Beispiel die Ausbreitung von Licht durch ein zufälliges Medium. Wenn wir die Wirkung dieses zufälligen Mediums in die einheitliche Entwicklung des Feldes durch das Medium einbeziehen, hängt der Impulsoperator, den man daraus ableiten würde, aufgrund der Zufälligkeit des Mediums von der Position ab.

Wenn der Impulsoperator nicht von der Position abhängt, spiegelt dies die Tatsache wider, dass das untersuchte System der räumlichen Translationsinvarianz folgt und daher die Erhaltung des Impulses unterstützt. In einem zufälligen Medium bleibt der Impuls nicht erhalten, da das Medium Lichtstreuung verursachen kann, was eine Änderung des Impulses impliziert.

Gleiches gilt jedoch für den Hamiltonianer. Wenn der Hamilton-Operator eine explizite Zeitabhängigkeit aufweist, ist das System in Bezug auf Zeitübersetzungen nicht unveränderlich, und die Energieeinsparung bricht zusammen.

Auf der fundamentalen Ebene wissen wir, dass sowohl Impuls als auch Energie erhalten bleiben. Dies spiegelt sich in der Tatsache wider, dass weder der Hamilton-Operator noch der Impulsoperator explizit von Zeit oder Position abhängen. Beispielsweise ist in Quantenfeldtheorien (wie QED) die Abhängigkeit von den Raum-Zeit-Koordinaten auf die der Felder beschränkt und erscheint im Lagrange nicht explizit. Die implizite Translationsinvarianz in Raum-Zeit-Koordinaten führt zu einem Noetherstrom, dem Energie-Impuls-Tensor, aus dem man die Ausdrücke für den Hamilton-Operator und die Impulsoperatoren erhält, die lediglich in Form der Felder und ihrer Ableitungen ausgedrückt werden.

Können wir als einfachstes Beispiel das Teilchen in einem Boxensystem betrachten? Hier kann der Impulsoperator nur in einem kleinen Intervall definiert werden (daher positionsabhängig), obwohl es andere Probleme hinsichtlich der Selbstadjunktheit des Impulses gibt, aber das ist ein anderes Thema von Interesse.

Denken Sie zuerst an den Hamiltonianer. Der Hamiltonianer ist ein Generator für Zeitübersetzungen, weshalb er eine Funktion von sein kann t t . Es kann zeitabhängig oder zeitunabhängig sein. Dies kodiert, wie sich das System als Funktion der Zeit entwickelt. Mathematisch gesehen ist der Impuls der Generator von Raumübersetzungen, so dass er im Prinzip eine Funktion eines Positionsparameters sein kann . Es kann also positionsabhängig oder positionsunabhängig sein. Dies würde uns sagen, wie sich das System im Raum entwickelt . Die meisten Experimentatoren machen keine Experimente, um Quantensysteme im Raum zu bewegen, aber sie machen Experimente, um Systeme in der Zeit zu bewegen. Es ist in Zukunft denkbar, dass ein Experimentator eines Tages einen Impulsoperator als Funktion eines Positionsparameters kennen müsste, wenn Techniken der Quantensteuerung weit fortgeschritten sind und Quantensysteme vor Dekohärenz geschützt werden können. Denken Sie auch daran, dass dies nicht relativistische Physik ist, Position und Zeit nicht auf dem gleichen Fundament stehen. Es gibt keinen Grund zu der Annahme, dass sie sich genauso verhalten sollten. Um Ihre Frage direkt zu beantworten, kann der Impulsoperator im Prinzip eine Funktion eines Positionsparameters sein.

Ich bin mir ziemlich sicher, dass ein Impuls, der eine Funktion der Position wäre, experimentell leicht beobachtbar wäre, da die Kommutierungsbeziehung zwischen Position und Impuls nicht gelten würde und Sie somit Verstöße gegen das Heisenbergsche Unsicherheitsprinzip beobachten würden, die wir einfach nicht beobachten.
Die Heisenberg-Kommutierungsrelation ist wahr, wenn der Impulsoperator unabhängig von einem Positionsparameter ist, aber dies sagt nichts darüber aus, ob es möglich ist, dass der Impuls von einem Positionsparameter abhängig ist. Es könnte durchaus eine Differentialgleichung geben, die in einem komplizierten System nach Impuls zu lösen ist, und solange der Impulsoperator hermitisch bleibt, bleibt der Operator, der auf den Quantenzustand einwirkt, der durch Exponentiierung des Impulsoperators erhalten wird, einheitlich und daher ein legitimer Quantenoperator.
Dies müsste überprüft werden, aber ich bin mir ziemlich sicher, dass jeder Hamiltonianer, der die Position auf ausreichend komplizierte Weise mit dem Impuls koppelt, Ihnen den Impuls als Funktion der Position geben würde.

Warum kann der Hamilton-Operator eine Funktion des Zeitparameters t sein, aber der Impulsoperator kann keine Funktion des Positionsparameters x sein?

Ein Impulsoperator kann eine Funktion von sein x x . Ich zitiere ausführlich aus den Seiten 57-58 von Aitchison & Heys "Gauge Theories in Particle Physics, 2nd Ed." Das ist also zu lang, um einen Kommentar abzugeben:

Der wesentliche Punkt ist, dass (etwa in einer Dimension) p ^ p ^ wird letztendlich vom Kommutator definiert ( = 1 ) ( = 1 )

[ x ^ , p ^ ] = i [ x ^ , p ^ ]] = ich

Sicherlich die vertraute Wahl

p ^ = - i x p ^ = - - ich x

erfüllt die Kommutierungsbeziehung. Wir können aber auch jede Funktion von hinzufügen x x zu p ^ p ^ und dies geändert p ^ p ^ wird da noch zufriedenstellend sein x x pendelt mit jeder Funktion von x x . Detailliertere Überlegungen von Dirac zeigten, dass diese beliebige Funktion tatsächlich die Form haben muss F. x F. x , wo F. F. ist willkürlich. Somit

p ^ ' = - i x + F. x p ^ ' = - - ich x + F. x

ist ein akzeptabler Impulsoperator.

Es gibt vielleicht zehntausend Operatoren, die die Kommutierungsrelation erfüllen, aber es gibt vermutlich nur einen Operator, der der Grenze von entspricht ich T. ^ ( x , x + Δ x ) - 1 Δ x ich T. ^ ( x , x + Δ x ) - - 1 Δ x wie Δ x Δ x geht zu 0. Ich bin daran interessiert, warum dieser Operator keine Funktion der Position sein kann.
@KeshavSrinivasan, fair genug, aber lassen Sie mich in Ihrer Frage auf Folgendes hinweisen: " Wenn also der Impuls eine Funktion der Position ist, würde dies die Positions-Impuls-Kommutierungsbeziehung beeinträchtigen."

Viele mathematische Antworten hier. Ich bin mir nicht sicher, wie zufriedenstellend sich diese "anfühlen" werden, da diejenigen, die ich gelesen habe, keine Intuition dafür zu geben scheinen, warum Momentum nicht die Ableitung der Position ist.

Ich denke, die intuitive Antwort ist einfach, dass der Impuls für eine Welle nicht der gleiche ist wie der Impuls für ein Teilchen. Dieser Impulsoperator ( ich d d x ich d d x ) ist eher ein "Wellenimpulsoperator" und misst den Impuls des Wellenpakets, das dem Quantenzustand zugeordnet ist. Aber nichts hindert Sie daran, den "Teilchenimpuls" zu verfolgen und explizit zu berechnen m d x d t m d x d t .

Hier ist eine anständige Antwort, um eine Intuition für den Impuls einer Welle aufzubauen:

Um das Problem zu klären, betrachten wir ein vereinfachtes Modell der Saite: Die Saite erstreckt sich entlang der x-Richtung und besteht aus Massen, die durch Federn verbunden sind. Nehmen wir zur besseren konzeptionellen Klarheit an, dass sich diese Massen nur auf und ab bewegen können (entlang y; dies könnte in einem mechanischen Modell erzwungen werden, indem die Massen auf kleinen Drähten auf und ab gleiten). In diesem Fall ist der mechanische Impuls eindeutig nur in y-Richtung (alle Bewegungen verlaufen entlang y), und es gibt keinen mechanischen Impuls in Ausbreitungsrichtung der Welle (x). Abhängig von der Form des Wellenpakets kann der Gesamtimpuls in y-Richtung auch Null sein, wenn sich einige Massen nach oben bewegen, während andere sich nach unten bewegen.

Dieser "Wellenimpuls" -Operator erweist sich als besonders nützlich, insbesondere bei der Beschreibung der Energie des Systems (und kann auch verwendet werden, um eine Bewegung in Position durch mehrmaliges Anwenden des Operators zu "erzeugen", aber diese Bewegung ist die Bewegung des gesamten Systems Wellenpaket).

Warum ist es nun speziell die Ableitung von x? Wir können eine gewisse Intuition gewinnen, wenn wir uns die ebene Wellenlösung für die Schroinger-Gleichung ansehen:

ψ ( x , t ) = e ich ( p x - E. t ) ψ ( x , t ) = e ich ( p x - - E. t )

Versuchen wir, nach p zu lösen und herauszufinden, was p spezifisch ist. Wir können das p dazu bringen, vom Exponential "herunterzukommen", indem wir eine partielle Ableitung anwenden.
wo p ist

ψ x = x e ich ( p x - E. t ) = i p e ich ( p x - E. t ) ψ x = x e ich ( p x - - E. t ) = ich p e ich ( p x - - E. t )

p ψ = ich x ψ p ψ = ich x ψ

Daher können wir schließen, was dieser p-Operator ist, indem wir alle Begriffe verschieben, um explizit zu zeigen, was p als Operator tun muss:

p ^ = - i x p ^ = - - ich x